How to Effectively Answer Law Essay Questions

I remember my first semester 100 level exams. I was just fresh from secondary school/JAMBITE, but I thought law exams were the same with secondary school exams. Heck, I didn’t even know that law exams were only theory questions. I was expecting to meet some objective questions, until I saw my exam questions.

Well, during the exams, I wrote what I could, and was confident my results would be awesome. After all, I had read for the exams well enough, and I used to think I was kind of intelligent.

I was in for a rude shock.

While I was in 100 level at the University of Ilorin, they still pasted everyone’s results on the notice board. So, when I heard the first result was out, I and a lot of my “fresher” colleagues went to check our results. I was expecting an A, or worse, a B.

I located my matric number on the pasted list and checked my first result. I had a C.

I was surprised, but I felt it was my first result, the others would be better.

The other results started trickling in. With each result pasted on the notice board, I realised I wasn’t so special after all. I had a series of C’s. For my 100 level first  semester results, out a total of 10 courses, I had just one A, two B’s and the rest were C’s.

I was dejected, along with most other “freshers” that received this glorious welcome to Faculty of Law, University of Ilorin.

Instead of blaming the poor results on the indiscretions of my lecturers, I knew something had to be wrong with what I wrote. So, I asked for help. I asked a scholar(the best student in a level) in 400 level at the time, Adekunle Charles , to show me how to answer law questions. He showed me the way, and I can tell you that my results improved dramatically.

So, I am going to teach you exactly what he taught me, how to answer law exam questions. I will be focusing on law essay questions in this post.

What are Law Essay Questions?

There are two major types of law questions, essay questions and problem questions. Law essay questions require you to write an essay. Unlike problem questions that require you to advise parties in a scenario.

We have all been answering a level of essay questions right from secondary school, so it shouldn’t be new to you.

The following is an example of a law essay question:

There have been a lot of arguments for and against the principle established in the popular case of Adams vs Lindsell . Expatiate, through the cases.

To answer law essay questions properly, it must follow four rules. It must have The Introduction, The definitions, the body, and the conclusion.

Answering Law Essay Questions Rule 1: The Introduction

The introduction to your law essay question is the part where you let the lecturer know what the answer is all about.

In this part of the question, you shouldn’t directly go into answering the question. Instead, you are allowed to beat about the bush a little bit. Start with a general statement and then become more specific. At the end of the introduction, you should talk about the law essay question you intend to answer.

As an illustration, this is how the introduction to the sample law essay question above should look like:

The importance of feedback in the formation of a contract cannot be over-emphasized. It is trite that every contract needs to have an offer and acceptance, and there is the need to communicate the offer and acceptance between the parties. In a lot of instances, this isn’t really a problem since the offer and acceptance is done in real-time (face to face). However, there are instances where it isn’t in real time, like when the communication is done by post. In this type of situation, due to the process of posting a letter or parcel, the communication between the parties can experience some delays. This has posed some problems, like “when is an acceptance valid?” Upon posting, or upon reception? One principle that has been developed by the courts to solve this problem is the rule in Adams vs Lindsell . This work is going to analyse this rule and talk about the criticisms levelled against it, with special attention being paid to case law.

Answering Law Essay Questions Rule 2: The Definition

This is the part of the question where you give a definition to the major terms/keywords in the question. It is not necessary that it has to be a “term” per se. For instance, in the sample question I gave above, the major term is Adams vs Lindsell .

So, what you should do at this stage is to define the rule in Adams vs Lindsell . Since this is a case, you should talk about the facts of the case.

Your answer can go something like this:

The rule in Adams vs Lindsell is generally referred to as acceptance by post. The rule in  this case was propounded by Lord Ellensborough in 1818. In this case, the defendant offered to sell some wool to the plaintiff. The defendant sent their offer by post. Due to an error in the posting, the letter got to the plaintiff on the evening of September 5. The plaintiff posted an acceptance the next day. If the letter was posted correctly, the defendant ought to have gotten the reply by September 7. So, when the defendant didn’t get a reply on September 7, he sold the wool to a third party on September 8. The plaintiff’s acceptance finally got to the defendant on September 9. Since the defendant had already sold the wool to a third party, the plaintiff sued for breach of contract. The major contention was when the acceptance would be valid. On the plaintiff posting it, or on the defendant receiving it. The court held in favor of the plaintiff that when it comes to contracts conducted by post, acceptance comes to fruition at the time of posting, not at the time of receiving.

Answering Law Essay Questions Rule 3: The Body

This is the major part of the answer to the law essay question. It is in this part of the answer that you demonstrate  your understanding of the question and knowledge of the subject matter. In a lot of instances, what differentiates an A student from a C student is the fact that an A student cited more authorities in this section of the answer.

Using the sample question above, this part of the answer to the law essay question will look something like this:

Since the inception of this rule, there have been numerous arguments for and against it by jurists, scholars, and judges alike. In the case itself, the court, in justifying its decision stated that if acceptance wasn’t complete on posting, then there is the need for the offeree to require the offeror to inform him that he had received his acceptance, and so it  goes on  ad infinitum . Scholars like Professor Sagay have disputed this justification of the rule in Adams vs Lindsell . According to him, the process doesn’t have to go on  ad infinitum.  The offeree can assume that a contract has come into fruition when the offeror receives the letter, the same way the the offeror has to assume that there is a binding contract when, and if, the offeree posts a letter of acceptance. In the subsequent case of  Household Fire Insurance Co vs Grant ,  the court gave some other concrete reasons for the adoption of the rule in  Adams vs Lindsell .  The facts of this case are as follows. The defendant applied for shares in the plaintiff company, and the plaintiff company assented by posting a letter. However, the letter didn’t get to the defendant, and as such, he didn’t know that the company accepted his offer. When the company got into liquidation, he was called upon to pay up his share. He resisted this, and thus the case was brought before the court. The court, in applying the rule in  Adams vs Lindsell ,  held  that he was liable to pay up his own shares, since a binding contract came into existence the moment the company posted its acceptance, regardless of the whether or not he received the letter. In justifying the acceptance by post rule, the court gave the following reasons: The post office is an agent of both parties. So, technically, a letter given to the post office is deemed communicated to  the offeror. By posting the letter of acceptance, there is already a valid and binding contract. There is no need for any other act to bring the contract to fruition. The offeree has merely assented to the offeror’s proposals. The offeror is free to make it a term of the contract that there  is no valid acceptance until he receives it. Any alternative rule would lead to fraud and delay in commercial transactions because the offeree would have to wait for confirmation from the offeror that he has received his acceptance. The rule is the most convenient compared to all other alternatives. However, the court’s decision was not unanimous. There was a dissenting judgement by Bramwell, L.J. He contended that if the basis of the rule was that it would cause hardship on the offeree, who might have already made arrangements based on the acceptance of  the contract, there is also hardship on the part of the offeror who might act on the belief that his offer was not accepted. This is even more relevant where the offeror didn’t receive the acceptance like in the present case. All this goes to show that the rule in  Adams vs Lindsell   isn’t one that enjoys unanimous consensus in the legal community. Recent decisions by courts in the United States suggest a shift away from this rule of acceptance by post. In the case of  Rhode Island Tool Co vs US F. Supp. 417 (1955) ,  the plaintiff’s made an offer to sell some bolts to the defendant. The defendant accepted by post, but the plaintiff discovered that they had quoted a very low price. To remedy this, they sent a telegram to the defendant revoking the offer. The telegram got to the defendant before the posted acceptance got to the plaintiff. The court held that the offer was validly revoked since the telegram got to the offeree before the plaintiff received the letter of acceptance. A similar thing happened in the case of  Dick vs US F. Supp 326 (1949) .  The facts of this case are quite similar to the facts in the above case. In this case, the offeree was the plaintiff and after accepting the offer by post, sent a telegram withdrawing it. The telegram got to the defendant before the letter of acceptance, and the court held that it was a valid revocation.

Answer Law Essay Questions Rule 4: The Conclusion

The conclusion to the law essay question is the final part of essay (just like the name suggests). There are two major ways you can conclude the essay: either by summarizing what you have written, or by giving a recommendation/comment.

To be on the safe side, you should just conclude by summarizing what  you have written. You should also make it clear that  you are concluding by including the phrase “In conclusion” at the beginning of the conclusion.

So, this is how the conclusion to the sample question would look like:

In conclusion, this work has highlighted the evolution of the rule in  Adam vs Lindsell  with special attention given to case law. This work highlighted the establishment of the rule, the justifications given by the court for this rule, and the criticisms against this rule. It finally showed a departure from this rule in other jurisdictions like the USA, due to the impact of new technology on commercial transactions.

Here’s the full answer to the essay question

So, this is how you should answer a law essay question. If you want to get a full picture of what the answer to the essay question looks like, here you go:

The importance of feedback in the formation of a contract cannot be over-emphasized. It is trite that every contract needs to have an offer and acceptance, and there is the need to communicate the offer and acceptance between the parties. In a lot of instances, this isn’t really a problem since the offer and acceptance is done in real-time (face to face). However, there are instances where it isn’t in real time, like when the communication is done by post. In this type of situation, due to the process of posting a letter or parcel, the communication between the parties can experience some delays. This has posed some problems, like “when is an acceptance valid?” Upon posting, or upon reception? One principle that has been developed by the courts to solve this problem is the rule in Adams vs Lindsell . This work is going to analyse this rule and talk about the criticisms levelled against it, with special attention being paid to case law. The rule in Adams vs Lindsell is generally referred to as acceptance by post. The rule in  this case was propounded by Lord Ellensborough in 1818. In this case, the defendant offered to sell some wool to the plaintiff. The defendant sent their offer by post. Due to an error in the posting, the letter got to the plaintiff on the evening of September 5. The plaintiff posted an acceptance the next day. If the letter was posted correctly, the defendant ought to have gotten the reply by September 7. So, when the defendant didn’t get a reply on September 7, he sold the wool to a third party on September 8. The plaintiff’s acceptance finally got to the defendant on September 9. Since the defendant had already sold the wool to a third party, the plaintiff sued for breach of contract. The major contention was when the acceptance would be valid. On the plaintiff posting it, or on the defendant receiving it. The court held in favor of the plaintiff that when it comes to contracts conducted by post, acceptance comes to fruition at the time of posting, not at  the time of receiving. Since the inception of this rule, there have been numerous arguments for and against it by jurists, scholars, and judges alike. In the case itself, the court, in justifying its decision stated that if acceptance wasn’t complete on posting, then there is the need for the offeree to require the offeror to inform him that he had received his acceptance, and so it  goes on  ad infinitum . Scholars like Professor Sagay have disputed this justification of the rule in Adams vs Lindsell. According to him, the process doesn’t have to go on  ad infinitum.  The offeree can assume that a contract has come into fruition when the offeror receives the letter, the same way the the offeror has to assume that there is a binding contract when, and if, the offeree posts a letter of acceptance. In the subsequent case of  Household Fire Insurance Co vs Grant,  the court gave some other concrete reasons for the adoption of the rule in  Adams vs Lindsell.  The facts of this case are as follows. The defendant applied for shares in the plaintiff company, and the plaintiff company assented by posting a letter. However, the letter didn’t get to the defendant, and as such, he didn’t know that the company accepted his offer. When the company got into liquidation, he was called upon to pay up his share. He resisted this, and thus the case was brought before the court. The court, in applying the rule in  Adams vs Lindsell,  held  that he was liable to pay up his own shares, since a binding contract came into existence the moment the company posted its acceptance, regardless of the whether or not he received the letter. In justifying the acceptance by post rule, the court gave the following reasons: The post office is an agent of both parties. So, technically, a letter given to the post office is deemed communicated to  the offeror. By posting the letter of acceptance, there is already a valid and binding contract. There is no need for any other act to bring the contract to fruition. The offeree has merely assented to the offeror’s proposals. The offeror is free to make it a term of the contract that there  is no valid acceptance until he receives it. Any alternative rule would lead to fraud and delay in commercial transactions because the offeree would have to wait for confirmation from the offeror that he has received his acceptance. The rule is the most convenient compared to all other alternatives. However, the court’s decision was not unanimous. There was a dissenting judgement by Bramwell, L.J. He contended that if the basis of the rule was that it would cause hardship on the offeror who might have already made arrangements based on the acceptance of  the contract, there is also hardship on the part of the offeror who might believe that his offer was not accepted. This is even more relevant where the offeror didn’t receive the acceptance like in the present case. All this goes to show that the rule in  Adams vs Lindsell  isn’t one that enjoys unanimous consensus in the legal community. Recent decisions by courts in the United States suggest a shift away from this rule of acceptance by post. In the case of  Rhode Island Tool Co vs US F. Supp. 417 (1955),  the plaintiff’s made an offer to sell some bolts to the defendant. The defendant accepted by post, but the plaintiff discovered that they had quoted a very low price. To remedy this, they sent a telegram to the defendant revoking the offer. The telegram got to the defendant before the posted acceptance got to the plaintiff. A similar thing happened in the case of  Dick vs US F. Supp 326 (1949).  The facts of this case are quite similar to the facts in the above case. In this case, the offeree was the plaintiff and after accepting the offer by post, sent a telegram withdrawing it. The telegram got to the defendant before the letter of acceptance, and the court held that it was a valid revocation. In conclusion, this work has highlighted the evolution of the rule in  Adam vs Lindsell  with special attention given to case law. This work highlighted the establishment of the rule, the justifications given by the court for this rule, and the criticisms against this rule. It finally showed a departure from this rule in other jurisdictions like the USA, due to the impact of new technology on commercial transactions.

So, here you have it, a guide to answering law essay questions. If you follow these guidelines, you should see an improvement in your grades. If you have any questions related to this, feel free to drop a comment.

P.S: If you are interested in an online course that makes it easy for you to get A’s in your law exams, you can check it out here:  Get Access to Ace LL.B Exams

72 thoughts on “ How to Effectively Answer Law Essay Questions ”

Nice one bro. But between cramming the materials given by lecturers or understanding the material which one might likely improved one’s chance of getting good grade in essay questions from your own experience.

In law exams, there are some things you have to cram. Things like the cases and statutes. For the explanatory part of the note, it’s best you understand it.

its so excited for me to find this most simple blog for law student as a guide, tnx so much, may the sky be your limit

Where can I find the statutes and cases book? I’m a political student

Thank you. This really helped Can you post the one of problem questions

Thank you so much for taking out time to be a silver linen in the dark clouds of a law student who now, understands better how to answer law questions. The time you took to practically explain this using the Adam V Lindsell case is not a waste. I duff my hat sir.

This is really useful and m gonna attempt this semester’s exams in this way. Thanks sir.

TThank you.Please I need tips on problem question.

I’m currently working on a blog post that covers that

Thank you.I will be glad if that is done in no time.

Wow!!! This is beautiful. Please what about problem question? My exam’s two weeks from today

I’m currently working on a blog post on how to deal with that. It should be out before your exams.

Where can i find it?

Here: https://djetlawyer.com/irac-how-to-answer-law-problem-questions/

wow! this is wonderful and great. this sample has taught me a better way of answering law essay question seriously. bro you are good honestly.

Thanks. It’s cool that I’ve been able to help.

Thank you very much sir. I have understood the format now. But however, if a topic that doesn’t have cases. Is it wise or necessary to find cases to relate to such topics. For instance, Legal reasoning in judicial process.

There are always cases for all topics. You just need to know where to find them. Besides, no case is specifically designated for a subject, you can use any case, as long as it is relevant.

Thanks bro, my inquisitiveness to studying law brought me to your site. Pls how do I make my dream come true. This is my nineteen years of secondary education. I have NCE N B. ed in pols n edu. Mgmt.

Eeeermmm… Have you applied to any university offering law?

Thank you so much. This has helped me greatly

You’re very welcome

Thanks bro, looking forward to that post on problem question. This is appreciated.

im inspired, this is really excellent, though it looks like a lot of work and memorization, but it was really helpful. Thanks

Thank you so much for this great Tips. have been reading it over and over again.

You’re very welcome. I’m glad to have been of help.

This is really helpful. Thanks and job well done.

Thanks a lot Mr Olamide More strength to your elbow

I am becoming more addicted to your blog, ‘barrister’ Olamide ? . Do not stop at anytime. Let’s keep flying

Thanks a lot bro.

Hello Olamide. Thank you for this comment but i really need your help for something person as regards to law. How can i reach you pls? Thanks

Send me an email.

Am very greatful for the advice you have given it’s really great. A concern: is it always a must to cite case laws when answering law questions? And what happens when you only remember facts oof the case and you don’t rremember the parties?

Cases and statutes are what separates the work of a law student from that of a sociologist or political scientists. It is quite essential that you try to cite case(s) or statutes when writing a legal piece, as they give it more authority.

It would be ideal if you remember all. But if you can’t you can just write “in a decided case”. This might not give you full marks, but you’ll still get something.

Thank you very much for the advice am really greatful

I am a law student in Ghana KNUST. It is great work you’re doing. Though most of your posts are Nigerian Law, the ones that are general is helpful to me. Thank you.

You’re welcome. I’m glad to have been of help.

A big thank you Barr Olamide this article was really helpful a lot of Law student doesn’t know how to answer law essay question but by the grace of God we will try as much as possible to adhere your tips…my regards

please, what website can I get access to full law cases from.

You can check lawpavilionplus.com. However, you have to pay.

Thanks What of problem question

Check this out https://djetlawyer.com/irac-how-to-answer-law-problem-questions/

Thanks for the advice But what if u are not so good in beating around the bush to make your answer look more interesting I usually just go straight to the point cause of time factor and I really can’t even do it.

The “beating about the bush” part is the introduction part of the answer. It should just be like one paragraph.

Thanks What of how to answer problem question

do you have to identify each section by the heading; For example 1. Introduction 2. Definition 3.Body 4.Conclusion in order to delineate these sections for the attention of the marker.

No, you don’t need to identify that. IDBC is just a framework you should have at the back of your mind when writing exams.

Please can you make a write up to writing a very good memorial for should I say guidlines to writing a very strong legal arguements. In relation to moot and mock. Thank you

Thanks for the feedback. I’ll consider it.

Am really impressed at you strive towards making a soft landing for prospective and present law student generally. In some instances,what if the question is not possessing this semblance for example “Discuss the duty of a counsel to the court” In the abovementioned question what will be the definition part?

And also if am asked to answer a short question like “the relationship between law and morality” do I still need the IDBC format?

Lastly,Is this format of answering questions only applicable in legal methods or it can serve all law essay questions?

I will be glad if you can answer me respectively…?

Thanks so much I’ve learnt a great deal, please can you post that of problem question? God bless you.

Good evening, Please I want you to out me through a law assignment (Principles of equity). The question is: ” Critically examine the contribution of equity to jurisprudence”

Thank you very much sir i have really learnt so much from your work today and i believe that before the end of this semester my grades will improve and also my knowledge and understanding on how to answer problem questions and law essay questions will improve. God bless you sir.

Please can I get your username on social media platforms…I would love to know you…you dont know how much this write up just helped me

I’m glad to have helped. I’m not so active on Social Media though. However, you can connect with me on linkedin here https://www.linkedin.com/in/olyray/ and twitter here https://twitter.com/olanrewajuolam6 . ALso, check my author bio for other social media platforms.

Okay..please can I get some materials on human rights..precisely regional protection of human rights…I can’t seem to permutate it to my satisfaction

I don’t think I currently have materials on that.

Pls can I get past questions for introduction to legal method and introduction to Islamic law

I don’t really have that at the moment.

zainab you are here…… wawu i never believed til now

Mr.Can you please help me with this: A was driving along Lagos Ibadan expressway on the 24th July, 2020 and he got to Interchange at about 5am, he then saw a BMW 2015 model under the bridge with a tag ‘for sale’. He called the no on the tag and bought the car at the rate of #500,000.00. two days after, as he was driving the vehicle on Lagos Island, he was stopped by the Police and was arrested for a stolen vehicle. He later located the seller who was also arrested by the police, he was released on bail and the vehicle was recovered from Mr A being a stolen vehicle. Mr. Intend to sue the seller for the refund of his money or to sue the police that he bought the goods in accordance with Sales of Goods Act. 1. Please advise Mr. A 2. Will your advise be different if Mr. A had bought the car at Ladipo Car Market?

Interesting and helpful will be waiting on problem question too.

Hello. Problem questions are treated here:

https://djetlawyer.com/irac-how-to-answer-law-problem-questions/

Thank you very much sir. It’s a blessings to cross paths with you going through this work. Question. Is it possible to just cite the case without giving facts of that case? For instance as was decided in Shaw v DPP 1962 AC 220 and then you continue with your analysis?

Yes. You can.

Good morning. I’m very new here. This was really helpful, thank you so much for these tips.

I however struggle with problem questions majorly. I would like to know if you have a post on how to answer problem questions??

Wow thank you so much sir. I have really been struggling with answering law essay questions This has enlightened me. I really hope I apply it well

Good morning. I’m very new here. This was really helpful, thank you so much for these tips.

Thank you very much sir . I have finally gotten a well explained answer on how to answer law questions I’m greatful 🙏

Leave a Reply Cancel reply

Notify me by email when the comment gets approved.

Join an online course that makes it easy for you to get A’s in your law exams, you can check it out here: Get Access to Ace LL.B Exams.

how to answer a law essay question

Answering Essay and Problem Questions

how to answer a law essay question

How A Good Lawyer Can Help You Obtain Disability Benefits And Get Your Life Back On Track

how to answer a law essay question

The Ugly Side of Online Matchmaking: How Trademark Infringements and Exorbitant Legal Fees Maintain Match Group’s Dominance

how to answer a law essay question

  • Tips for Students
  • essay writing
  • Law Student
  • problem questions
  • tips for students

how to answer a law essay question

Article written by Imaan Fatima, University of Strathclyde Scots and English LLB Student.

There are two main types of questions you will come across when studying law: essay questions and problem questions. Both styles of questions can cover any area of law but require very different approaches when being answered. Every university will have a different grade criterion, so it’s important to become familiar with yours. However, there are some general tips which can be used by all law students to help best answer these questions.

Essay Questions:

I’m sure most of us have encountered these types of questions. We’re usually asked to critically evaluate, compare, or discuss some sort of legal issue.

These questions require a structured answer with an introduction, a main body, and a conclusion. But before you even attempt to produce an answer, you need to create a plan. This is crucial because it will help you understand what is being asked, set out the key issues you wish to discuss and the sources you want to use.

Your introduction should be relatively short – roughly 10% of the overall word count. It should clearly outline what the question is asking you to do and how you propose to go about answering this question to arrive at an appropriate conclusion. The main body of your essay will consist of legal arguments. Each paragraph will likely answer a different aspect of the question, but they should be coherently ordered and flow into one another, instead of reading as a sequence of unrelated points. Finally, you’ll have to draw together your arguments, without introducing any new pieces of information, to reach a reasoned conclusion.

When writing an essay, think about what the examiner is looking for and the skills that are being tested: communication skills; analytical skills; research skills; and the ability to construct an argument.

Problem Questions:

Unlike essay questions, problem questions are presented as lengthy scenarios in which one or more legal disputes arise. As a law student, you will be expected to discuss these disputes and explain how they are likely to be resolved, if possible.

The best approach to answering problem questions is the IRAC method, which stands for issue, rule, application, and conclusion. This is a four-step process that requires you to identify the legal issue; identify the relevant legal rule; apply that rule to the issue at hand; and reach a conclusion to answer the question. It’s important to remember that the IRAC method should never be applied to the question as a whole. Instead, your answer should be divided based on the number of separate legal issues which occur.

Problem questions often have no single correct answer, and the facts will likely contain a number of grey areas, which means you might not always have enough facts to provide a conclusive answer. However, you can also gain marks by stating that some information is missing and hypothesising what the outcome would be if certain facts were included.

Whereas essays are designed to test your ability to discuss and critically evaluate the law, problem questions are more concerned with the application of legal principles. This means simply having knowledge of the law isn’t enough. Neither is writing everything you know about the topic and hoping to receive marks for it. You need to be able to identify relevant information within the question and apply legal reasoning to the facts.

Imaan Fatima

Imaan Fatima

Related posts.

how to answer a law essay question

8 Tips for Finding the Best Truck Accident Lawyer for Your Case

how to answer a law essay question

California’s Laws Regarding Pain and Suffering Damages: Compensation for Non-Economic Losses

how to answer a law essay question

AI ethics or AI Law concept.bias in machine learning algorithms.Developing AI codes of ethics. Compliance, regulation, standard , business policy and responsibility for guarding.

Exploring the Use of Artificial Intelligence Software for Law Students

Comments are closed.

IPSA LOQUITUR

Writing Law Problem Questions

How to write a legal problem essay.

Law problem question essays give you an imaginary scenario. They then ask you to comment on the legal issues that arise and advise the parties. This guide will explain how to answer a problem essay with eight handy tips.

1. Read the Facts

The first step to answering any law problem question is to read the entirety of the facts you are given. Do not just jump into answering the question. Take your time and ensure that you fully understand all the issue involved in the case. It may help to highlight parts of the fact-pattern that you think are important.

2. Structuring Your Analysis

This is a mistake many law undergraduates and GDL students make when writing a scenario law essay. They look at the facts and recognise that they are similar to a case they’ve read. They will then immediately assume ‘this is like Joe Bloggs vs John Smith’, and answer the question accordingly. For this reason, examiners often complain that law essays lack coherent structure or proper analysis.

To avoid this pitfall, when answering legal problem questions you must adopt a structure .

Say you have an problem on whether or not the defendant is liable in the tort of negligence. The facts look similar to a particular case you’ve read on contributory negligence. Your first instinct is to start talking about defences. Stop . Ignore the similarity completely for now, and think. Before you can even discuss defences, you must talk about whether the defendant is liable in the first place. What does the law actually require you to establish to prove liability? Is the defendant liable? If so, what defences might he rely on, and how are those established?

Develop steps that you can put every scenario relating to that area of the law through in order. For example:

Contract Law

Contract law, signing agreements

  • Has there been an offer ?
  • Was the offer accepted ?
  • Are the terms of the agreement certain ?
  • Do the parties intend to be legally bound ?
  • Is there consideration ?
  • What are the terms of the contract?
  • Is there a breach ?
  • What kind of breach has occurred?
  • Is there a defence to the breach?
  • What are the innocent party’s remedies and options?

Criminal Law

Criminal law, holding a fence

  • Is the actus reus of the offence established?
  • Is the mens rea of the offence established?
  • Is there a relevant defence?

Negligence Law

Negligence law, stack of cars

  • Does the defendant owe the claimant a duty of care ?
  • Has the duty been breached ?
  • Is the breach a factual cause of the loss?
  • Is the breach a legal cause of the loss?
  • Is the loss sufficiently non-remote ?

This doesn’t mean you have devote a whole paragraph to every step. If its obvious that the defendant owed a legal duty, a single sentence pointing this out will suffice. If the question tells you there is a contract, simply note you will assume the contract was validly-formed. The most controversial and difficult points should be given the majority of your essay’s attention.

Nevertheless, you get credit for completing each necessary step in the order in which they arose. This is how a court of law would most naturally consider them. This method of structuring essays also stops you missing interesting elements of problem questions. You might miss these points by skipping straight to what you think is the most obvious issue.

3. Structuring the Overall Essay

The problem question may present you with multiple areas of law. For example, a defendant may have committed different kinds of crime, or liability might arise in both negligence and nuisance. Alternatively, the question may ask you to discuss the liability of several different people.

If you have to advise different parties, discuss the liability of each party in turn . Otherwise, a good rule is to  deal with each area of the law in turn . Give each area of law its own separate section. This will lead to a clearer essay structure than trying to deal with each event chronologically.

4. Structuring Individual Paragraphs

You can structure your discussion of individual issues any way which makes sense and follows a logical structure. One of the most popular ways for structuring discussions is the IRAC method. This stands for:

State the issue you are about to discuss

State the applicable legal rules (with authority or statutory references)

Discuss how these legal rules apply to the facts in front of you

State what the conclusion to the issue is based on your analysis

For example, lets say you are writing a problem question in tort law. You have a defendant who has run over a pedestrian with his car. You think he may be liable in negligence, so you start by considering whether he owes a duty of care. You would structure your analysis of this issue as follows:

The first issue is whether the defendant owes the claimant a duty of care.

If the case’s facts are non-novel, whether a duty is owed depends on the applicable precedent ( Robinson v CC of West Yorkshire Police ). It is established that road-users owe others a duty of care ( Nettleship v Weston ).

In this case, the defendant is a road-user because he was driving a car on the road. The claimant was also a road-user because they were a nearby pedestrian.

Therefore, the defendant owes the claimant a duty of care.

5. Not Enough Information Given?

What if the problem question does not seem to give you enough information to advise on the parties’ legal position? It is completely fine to write ‘the problem question does not give us enough information to determine X’.

For example, lets say you think that whether the defendant was in breach depends on how fast he was driving. The facts do not tell you how fast he was driving. Do not be afraid to say so!

However, saying that there isn’t enough information is NOT enough. You must then go on to say what information you would need to advise the party. Once you have set this out, explain how the law would apply to the facts if you had this information.

For example, you might say ‘if the defendant was speeding, he is likely in breach of his duty.’ Explain why. Then, say that ‘if he was not speeding, he is likely not in breach.’ Finish by explaining why this is the case.

This shows you are thinking like a lawyer. If a client walks in and gives half the facts, you need to be able to ask the right questions. You then need to be able to evaluate the facts, whatever they turn out to be.

6. Red Herrings

Sometimes, the fact-pattern will include information which seems salient, but actually is not. This is known as a red herring. These red herrings are designed to test your ability to pick apart relevant information from irrelevant information.

Many students assume that all of the information in the problem question must be relevant and addressed. When presented with a red herring, these students will panic and start guessing at how the information affects the outcome. Instead, the examiners expect you to point out that the information is not relevant. You can get bonus points by explaining why the law doesn’t consider this kind of information relevant.

For example, say you have a commercial law question where the owner of a painting stores it with a local art dealer. The owner then sees that the art dealer has put the painting up for sale without authority. The owner decides he will call the dealer later in the day to clear up the problem. However, he negligently forgets. The painting is later sold to a third-party.

The red herring here is the owner’s negligence. This seems like it should be relevant to whether the owner has lost property in the painting. In reality it is not: you cannot lose ownership of property because you were negligent. Because the information seems relevant, it is tempting to discuss it at length, for example by talking about estoppel by negligence. This wastes time and will not get you any marks. Rather, you should say ‘the owner’s negligence is not relevant to whether he retains ownership of the painting’. Then, move on.

7. Don’t Hedge Your Bets (and other Stylistic Tips)

If there’s one thing most law professors hate, its a phrase like ‘it seems from the evidence that there might be a possibility of supporting the argument that…’.

Confidence in essay-writing is not something that is stressed enough at school or university. When you aren’t sure, it is tempting to hedge your bets with language like ‘probably’ and ‘it might be the case’. Resist that urge. If your analysis is correct but don’t sound confident, the examiner may doubt that you firmly grasp the material. If it is not correct, saying ‘probably’ in front of the error won’t help in any case.

Other stylistic tips for writing a professional sounding essay include:

  • Avoid contractions (‘don’t’, ‘can’t’), slang phrases and other informal language;
  • Avoid the phrase ‘it is submitted that’. This kind of wording is for moots and legal debating, not academic legal essays;
  • Try to deal with only one issue per paragraph. This makes the essay less visually intimidating;
  • If simple language and short sentences get your point across, use simple language and short sentences. There is a temptation to sound ‘professional’ by using multi-clauses sentences and complex vocabulary. This just makes the essay harder to read.

8. Cite, Cite and Cite Again

If you ever make any kind of positive claim about the law, back it up with a citation. What proves your claim? A case? A statutory provision? Cite it. You need to assure the marker that you aren’t just making lucky guesses. Also, many institutions’ grading criteria specify that you can’t get more than a 2:2 if there is insufficient citation.

Generally there is no need to give the year, report and page number of case-law in exams. However, you should check your university’s best practice guidelines to know for sure.

Share this:

how to answer a law essay question

  • 0 Shopping Cart $ 0.00 -->

JD Advising

Sign up for our FREE comprehensive “How to Pass the Bar Exam” 4-day mastery class . Your journey to success begins here!

Jumpstart Your Law Career! Exclusively for incoming law students, JD Advising’s Scholars Program offers 10 hours of personalized tutoring, essential study aids, and networking with top law firms. Transform your potential— apply for free today !

🌟 Boost Your Bar Exam Prep with JD Advising! Looking for one-on-one attention in your bar prep? Our expert bar exam tutors are ready to help you pass! Sign up early before spots fill up!

how to succeed in law school, excel in law school, excel 1L year

How do I answer law school exam questions? (an in-depth guide)

Here, we give you an in-depth guide on how to answer law school exam questions. This is a step-by-step process for approaching law school exams.

Law school exams are very different than exams you may have taken in undergrad . Because they are different from the exams you are used to, you need a novel strategy if you want to answer the law school exam questions successfully and receive high scores on your law school exams. Many students do not know how to best prepare for law school final exams (in part because law schools do not teach students this!). In this post, we break down the process of answering law school exam questions into a few simple steps.

How to answer law school exam questions

Preparation for answering law school exam questions.

Make your outlines!

Before you learn  how  to answer law school exam questions and attempt to answer law school exam questions, it is crucial that you have outlines (that you have made) and that you have familiarized yourself with your outlines well enough so that you feel like you know them. Here we have an in-depth guide to outlining . We also have tips on how to outline using diagrams , and places you can find outlines online if you want to review examples of outlines.

Learn your outlines!

If you have outlines, but are struggling to learn or memorize them, you are not alone. Many students struggle with this aspect of studying. Check out this post for seven fantastic tips on learning your law school outlines, which is outlined previously in this guide!

Why do you need to outline and learn your outlines first? Because the essential tool that you need prior to answering exam questions is knowledge of the law! This is the key tool, or your secret weapon, that will help you work your way through exam questions. After all, exam questions will ask you to apply the law to a fact pattern . You cannot apply the law if you do not even know what the law is! No strategy, tool, or tip will be able to get you a high score on a law school final exam if you do not make an effort to truly know the law.

General format of a law school exam

Your law school exam will generally be a story (called a “fact pattern”). You will be expected to go through the story, sentence by sentence, and spot the issues. For example, you may have a fact pattern like this:

Anna is walking down the street, texting her friend. She is not looking where she is going. A driver, Bob, who is trying to get directions on his phone, does not see Anna and hits her. Anna falls to the ground and breaks her tailbone. Normally, the impact would not cause a broken tailbone but Anna has a brittle bone disease which causes her to get hurt very easily. Anna goes to the doctor later that afternoon. Anna fills out a form that clearly states she is allergic to various medicines. The doctor accidentally gives her medicine that she is allergic to—a medicine that Anna had listed on her form that she was allergic to. Anna has to stay in the hospital for five days to be treated for her allergic reaction. The visit for the broken tailbone cost Anna $20,000. The five-day stay in the hospital cost Anna $50,000. Discuss the issues. 

Your job will be to use the IRAC method, discussed below, to identify the issues, state the rules of law, apply them, and then arrive at conclusions for each issue spotted. You can see that the call of the question is open-ended (“Discuss the issues”) and that you are not actually told which issues to discuss. Note that your professor may give you several short fact patterns (like the one above) or may give you longer fact patterns (sometimes one to five pages long).

Step one: learn to answer exam questions using the IRAC method

The IRAC method is a commonly used method for writing law school exam answers. The letters in IRAC stands for the following words:

A = analysis

C = conclusion.

The IRAC method is formulaic. To use IRAC one essentially fills in the blanks of the following formula for each issue that one spots: “The issue is ___________. The rule is ___________. A court would analyze it as follows: [apply the law to the facts]. In conclusion, ___________ will likely win.”

The IRAC method, in its most basic form, is not the best way to answer exam questions. It is a bit too simplistic and formulaic. However, it is a great start for learning how to answer law school exam questions. Further, as you practice it, (and as we show you below) you will find yourself improving the analysis section significantly in order to write a sophisticated exam answer.

It is important that you identify most, if not all, of the issues in your exam fact pattern.  Write down the issue. Explicitly say, “The issue is whether . . . .” Note that many times, the fact pattern will not give you any hints as to what the issues are.  It will just say “Discuss the issues” or “Discuss.”  Thus, it will be up to you to spot all of the issues.

Issue-spotting is important because if you don’t spot the issue, you will not be able to apply the relevant law and analyze it (and this is what you get the most points for!). To become a good issue-spotter, practice answering exam questions and comparing them to model answers.  This will help you get an idea of the most commonly tested issues and will help you spot these issues on your final exam.

As an example, in the example fact pattern above, some of the issues are negligence, contributory negligence, foreseeability of medical malpractice, eggshell-skull rule, and joint and several liability. All of these issues should be discussed separately.

For each issue that you identify, state the rule of law that governs the issue. Make sure that you state the relevant rules of law.  Some students will write down all of the rules of law that they have learned to try to impress the professor. However, you don’t get points for these and you are wasting time writing them all down.

The analysis is the most important part of your exam answer. It is the section where you apply the law to the facts. The key to writing an average IRAC answer and an outstanding A+ answer is to develop the analysis by making lawyerly arguments on behalf of the plaintiff and on behalf of the defendant for each issue that you spot. When you get to the “A” in IRAC, ask yourself: “What would the plaintiff argue?” then ask yourself, “What would the defendant argue?” State who has the better argument. Make creative arguments, and make all of the reasonable arguments you can. This is the most important part of your essay.

When you make arguments (and when you respond) for each side, refer to specific facts in the fact pattern. Arguments can be made using the following strategies:

  • Sometimes you will argue that the facts should be interpreted one way or another.
  • Sometimes you will argue that the law should be interpreted one way or the other.
  • Sometimes you will cite policy reasons for why the case should turn out one way or another.
  • Sometimes you will argue that the traditional rule should be applied instead of the model rule (or vice versa) or that the common law rule should be applied instead of the statute (or vice versa), or that the majority rule should be applied instead of the minority rule (or vice versa).
  • Sometimes you will argue that the holding in one case should be followed rather than the holding in a different case.

Most of your arguments will be based on classroom discussion. That is, if your professor really emphasized contradicting rulings of two different cases, you would mention that in your argument section. If your professor focused instead on common law rulings vs. statutory law, you may make arguments based on that.

Even if the question says “you are counsel for the defense,” you still have to recognize (and state in your answer) arguments that the plaintiff would make. After all, the best defense lawyer will anticipate arguments by the plaintiff.

This takes your answer out of the simple IRAC format because you are not just applying the law to the facts, instead you are arguing and analyzing from both the plaintiff’s and the defendant’s perspective in a lawyerly manner.

Last, state which party is more likely to win and explain your reasoning. The conclusion answers the question, “Who has the better legal argument?” The conclusion that you arrive at is not as important as the analysis that you provide.

Do not leave your conclusion too inconclusive (e.g., “It depends on what the court decides.  Either could win.”).  A conclusion that is inconclusive is not a conclusion at all! However, do not leave your conclusion too decisive either (e.g., “The plaintiff will definitely win” or “The defendant’s argument is baseless.”).  It is best not to use extremely strong words or phrases unless they truly are appropriate (and this will happen very rarely for the big issues!). The keywords to use are “probably” or “most likely.” Say something like, “For all the reasons discussed above, the defendant is most likely to win on this issue.”

Step two: practice exam questions

It is not enough to have a good strategy for answering exam questions. You have to also practice that strategy . If you were trying to get good at painting, reading books about painting all day wouldn’t be enough—you would actually have to practice painting! It is the same with answering exam questions. It is not enough to have some skills or strategies. Instead, you have to practice those skills and strategies.

The three primary sources that you should use to practice applying what you know are:

  • (1) Exams that your professor has given students in the past (check with your professor) ***This is by far the best resource!*** Make sure to obtain model answers or sample answers if possible.
  • (2) Problems in supplements— such as Examples and Explanations or Glannon Guides . This is a great resource for practicing all throughout the semester because you can practice analyzing one issue at a time and hone in on your skills.
  • (3) Other resources that contain exams with model answers:  The best thing to do is Google, for example, “Contracts exam model answer.” You will find updated Contracts exams with model or sample answers. We also have compiled a great resource of practice exams here if you want to save yourself some time Googling it!

After you write your exam answers, always spend a long time comparing your answer to the model answer and grading your answer. This is how you will improve on your exam-writing skills. You will be actively learning the law, giving yourself feedback, identifying your strengths and weaknesses, and find yourself improving over time! If you do this regularly, by the time you get to the final exam, you will know what to expect, and it honestly won’t seem like that difficult or daunting of a task because you will have already done it so many times! I graduated as the #1 law student and got A+’s in over half of my law school classes. Practicing exams ahead of time was a huge reason  I was able to achieve that accomplishment.

If you are unsure how to compare your answer to the model answer, we recommend that you start by asking yourself these questions:

Questions to ask when comparing your answer to the model answer

  • Did I spot the same issues that the writer of the model answer spotted?
  • Did I miss important issues? Which ones? How will I avoid this in the future?
  • Did I include issues that the model answer did not include? If so, are these issues relevant?
  • Did I clearly lay out all the rules and elements of law for each issue?
  • Did I discuss laws that weren’t relevant? How can I avoid this in the future?
  • Did I make arguments on behalf of each party (where applicable)?
  • Did I analyze the problem as in-depth as the model answer did?
  • Did I spend too much time analyzing an issue that should obviously turn out in one party’s favor?
  • Did I know enough law to fully analyze the question or do I need to review my outline more?
  • Was my conclusion too vague? Was it too strongly worded?
  • Was my conclusion correct (or at least arguably correct)?
  • Did I spend too much time restating facts or conclusions?
  • Did I answer the exam in the appropriate amount of time?
  • What are my strengths?
  • How can I capitalize on my strengths and make them even better?
  • What do I need to practice more?

Commonly-asked questions on how to answer law school exam questions

What is the most important resource i can consult if i have limited time.

The best resource at your disposal is your professor’s old exams. These show you what the professor likes to test and can help you get used to their format and their exam approach. Make sure that you obtain any model answer or sample answer that goes with the exam. Some professors do not make past exams available for review. Some only offer a couple of prior exams available for review, in which case it is a good idea to seek exams using the other resources mentioned above.

What if my professor includes multiple-choice or short answer questions?

It is becoming more common to include a multiple-choice question section and/or a short answer question section on law school exams. Below we tell you how to answer law school exam questions that contain multiple-choice, short answer, or other nontraditional questions.

Multiple-choice

If your professor includes multiple-choice questions on your exam, it makes sense to get as much practice answering multiple-choice questions as you can before your exam. Do a Google search for questions or, if you have time, order books online with multiple-choice questions. Note: we also have hundreds of law school multiple-choice questions in our JD Advising study aids (which you can sample for free!) Many bar exam review books will have multiple-choice questions that cover the material in first-year law school courses.  It is amazing how much you can increase your score through practice!

Make sure that when you begin to practice answering multiple-choice questions that you go through the questions slowly and methodically. Dissect each question. After you read the question, ask yourself what legal issue is being tested and what legal rule you need to know to answer the question. If you do not know the legal rule, look it up in your outline. Lastly, go through the answer choices and explain why one is correct and why the other three are incorrect. If you complete all of the questions slowly and methodically when you practice, you will learn the legal rules better and you will also internalize the best way to approach questions on your exam. You will not fall for tricks!

Closer to your final exam, work on speed. Practice answering several questions each day and make sure you are able to answer them in the time allotted.

If you are looking for a lengthier article about how to improve your score on multiple-choice questions, see this post on how to answer multiple-choice questions : it is geared toward bar exam students but can be helpful to any students taking a multiple-choice exam!

Short answer

If your professor includes short answer questions on his or her exam, get your hands on as many short answer questions as possible. If your professor releases any past exams, focus on these first! Also, consult a supplement. Many supplements have plenty of short answer questions for you to answer. (Our law school study aids contain hundreds of short-answer questions written by top law professors!)

Combination

If your professor gives a combination of questions (some multiple-choice, some essay) spend your time answering both kinds of questions when you practice. Allocate your time based on how your final exam score is calculated. If, say, only 10% of the exam is based on your multiple-choice score, and 90% is based on your essay score, focus primarily on practicing essay questions. If it is the opposite, focus primarily on practicing multiple-choice questions!

Go to the next topic, Where can I find law school practice exams?

Seeking success in law school.

  • Benefit from personalized one-on-one tutoring by our seasoned law school tutors.
  • Explore our NEW and highly acclaimed law school study aids , available for a free trial.
  • Introduction
  • How to Succeed in Law School: Introduction and Table of Contents
  • An Overview of How to Succeed in Law School
  • Part 1: 1L Basics – What is 1L year?
  • Overview of 1L year: what classes will I take my first year of law school?
  • Why does the first year of law school matter so much?
  • What are the differences between the first year of law school and college?
  • What law school supplies do I need to succeed in law school?
  • Just for fun! Top 10 Latin words for law students
  • Part 2: How to Succeed on Law School Final Exams
  • Law school success: the theory
  • An overview of how to succeed on law school final exams
  • Step 1: Outline
  • How do I create a law school outline? (An in-depth guide)
  • When should I start my law school outlines?
  • How long should my law school outline be?
  • Where can I find law school outlines?
  • Step 2: Memorize your outlines
  • How do I memorize my law school outlines?
  • Step 3: Take practice exams
  • What are law school finals like?
  • How do I answer law school exam questions? (An in-depth guide)
  • Where can I find law school practice exams?
  • Step 4: Make an effective law school study schedule
  • What should my weekly law school study schedule look like?
  • What is an example of a daily law school study schedule checklist?
  • Part 3: Other Important Law School Skills
  • What are “cases” in law school?
  • How do I brief a case?
  • What is an example of a law school case brief template?
  • How do I speed-read cases in law school?
  • How to prepare for class in law school
  • Why is it so important to go to class in law school (and pay attention!)?
  • How do I prepare for class in law school (without wasting time)?
  • What are some law school note-taking tips and shortcuts?
  • Socratic Method
  • What is the Socratic Method in law school?
  • How do I survive the Socratic Method in law school?
  • Networking in law school
  • Is it important to network in law school?
  • Part 4: Law Student Frequently Asked Questions (FAQ) and Other Resources
  • Law student FAQ
  • Is a law school tutor worth it?
  • What are some other JD Advising resources for 1Ls?
  • Download Guide

After working with [my law school tutor], Jonathan, I earned grades high enough to land a summer associate position at a big law firm as a 1L. Jonathan was great because not only was he well versed in my 1L courses, but he also is a successful lawyer and a graduate of my law school. It was great being able to talk to someone who not only understood the rigor of my 1L classes, but also had proven tips for success in and beyond law school.

My [law school] tutor Melissa was very flexible to my needs and interests. She helped me achieve Honors grades in three of my four doctrinal courses of my 1L Fall semester. Very grateful for this opportunity.

  • Privacy Policy
  • Terms of Use
  • Public Interest

September Bar Exam, Whether To Be A Lawyer

By using this site, you allow the use of cookies, and you acknowledge that you have read and understand our Privacy Policy and Terms of Service .

Cookie and Privacy Settings

We may request cookies to be set on your device. We use cookies to let us know when you visit our websites, how you interact with us, to enrich your user experience, and to customize your relationship with our website.

Click on the different category headings to find out more. You can also change some of your preferences. Note that blocking some types of cookies may impact your experience on our websites and the services we are able to offer.

These cookies are strictly necessary to provide you with services available through our website and to use some of its features.

Because these cookies are strictly necessary to deliver the website, refusing them will have impact how our site functions. You always can block or delete cookies by changing your browser settings and force blocking all cookies on this website. But this will always prompt you to accept/refuse cookies when revisiting our site.

We fully respect if you want to refuse cookies but to avoid asking you again and again kindly allow us to store a cookie for that. You are free to opt out any time or opt in for other cookies to get a better experience. If you refuse cookies we will remove all set cookies in our domain.

We provide you with a list of stored cookies on your computer in our domain so you can check what we stored. Due to security reasons we are not able to show or modify cookies from other domains. You can check these in your browser security settings.

We also use different external services like Google Webfonts, Google Maps, and external Video providers. Since these providers may collect personal data like your IP address we allow you to block them here. Please be aware that this might heavily reduce the functionality and appearance of our site. Changes will take effect once you reload the page.

Google Webfont Settings:

Google Map Settings:

Google reCaptcha Settings:

Vimeo and Youtube video embeds:

You can read about our cookies and privacy settings in detail on our Privacy Policy Page.

JD Advising

  • Privacy Overview
  • Strictly Necessary Cookies

This website uses cookies so that we can provide you with the best user experience possible. Cookie information is stored in your browser and performs functions such as recognising you when you return to our website and helping our team to understand which sections of the website you find most interesting and useful.

Strictly Necessary Cookie should be enabled at all times so that we can save your preferences for cookie settings.

If you disable this cookie, we will not be able to save your preferences. This means that every time you visit this website you will need to enable or disable cookies again.

how to answer a law essay question

Public Law for Everyone

by Professor Mark Elliott

Writing a Law essay? Remember to argue!

Providing advice in the abstract about how to write Law essays is difficult because so much depends on the nature of the question you are answering. It’s also important to take into account whatever are the expectations for your particular course, degree programme or university. Nevertheless, a useful rule of thumb, I think, is that a good Law essay will normally set out and advance a clear thesis or argument . (Note that I’m referring here to essays as distinct from problem questions: the latter call for a different approach.)

The need for an argument

Some answers explicitly call for this. Take, for example, the following essay title:

‘Do you agree that parliamentary sovereignty is the most important principle in the UK constitution?’

Here, the question itself in effect advances an argument — that parliamentary sovereignty is the most important principle in the constitution — and invites you to say whether you agree with it or not. And in saying whether you agree, you need to advance your own argument: ‘I agree with this because…’. Or: ‘I disagree because…’. Or even (because if the question advances a position that you think implies a misconception, oversimplification or false premise, you can say so): ‘I will argue that the question oversimplifies matters by assuming that a particular constitutional principle can be singled out as uniquely important…’

Other questions may indicate in a less direct way the need for you to put forward your own argument. For example:

‘“Parliamentary sovereignty is the most important principle in the UK constitution.” Discuss.’

Here, we don’t have a ‘do you agree?’ prompt; instead, we have the apparently less directive ‘discuss’ prompt. If we read the question literally, it may seem that there is no need for you to put forward your own argument here. After all, it’s possible to ‘discuss’ something without advancing your own argument about it: you could make various points, explain various matters, and leave the reader to make up their own mind. But while this may be formally true, it’s unwise to read the question in this way, because it creates the risk that you will end up writing something very general and descriptive on the topic without going any further.

To summarise, then, there are at least three reasons for making an argument part of your essay. First, the question will often call for this, whether explicitly or implicitly, such that you wouldn’t be answering the question if you didn’t set out and develop an argument. Second, if you don’t impose on yourself the discipline of articulating and defending an argument, you risk underselling yourself by writing something that is descriptive and meandering rather than purposefully constructed . Third, setting out and developing an argument involves taking ownership of the material. By that, I mean using the material in a way that serves the purposes of your argument, showing that you are in command of it and that it is not in command of you. This, in turn, provides an opportunity to demonstrate a level of understanding that it would be hard to show in a descriptive essay that simply wandered from point to point.

Setting our your thesis

If putting forward an argument is (often) important or necessary, how should it be done? There are no great secrets here: the formula is straightforward. You should begin your essay by stating your thesis — that is, by setting out what it is that you are going to argue. This should be done in your introductory paragraph — by the time the reader reaches the end of that paragraph, they should be in no doubt about what you are going to argue. Imagine, for instance, that you are presented with the following essay title:

‘“The courts have expanded their powers of judicial review beyond all acceptable constitutional limits in recent decades; it is time to clip the judges’ wings.” Discuss.’  

In response to such a question, it might be tempting to say in your introduction that (for example) you are going to ‘show’ how the courts’ powers of judicial review have grown, ‘consider’ why this has happened and ‘examine’ the criticisms of judicial over-reach that have resulted. These are all perfectly sensible things to do when writing an essay on this topic, but if that is all you say in your introduction, you will leave the reader wondering what you think — and what you are going to argue . In contrast, an introductory paragraph that lays the foundation for essay that properly advances a thesis will set out what that thesis is. You might, for instance, take each of the propositions set out in the question and stake out your position:

‘In this essay, I will argue that (a) while the courts’ powers of judicial review have grown in recent decades, (b) it is misguided to suggest that this has breached “all acceptable constitutional limits” and (c) that those who now advocate “clip[ping] the judges’ wings” misunderstand the role of the judiciary in a rule of law-based constitution. In other words, the courts’ judicial review powers are entirely appropriate and those who seek to limit them risk undermining the rule of law.’  

An introduction of this nature would achieve two things. First, it would make clear to the reader the position you proposed to take. Second, it would immediately lend the essay a structure.

Developing your thesis

Once you have set out your thesis in the introduction, you need to develop or defend it. This will involve making a series of connected points in successive paragraphs, each of which relates to your overarching thesis. One way of thinking about this is that the individual points you make in the main body of the essay should all relate or point back in some way — and in a clear way — to the position that you staked out in the introduction.

In the example introduction above, the overarching thesis is set out in the second sentence; the individual and connecting parts of the argument are set out in propositions (a), (b) and (c) in the first sentence. One approach, therefore, would be to divide the answer, once the introduction has been written, into three parts, dealing in turn with points (a), (b) and (c). Naturally, as you work through the various parts of your argument, you will need to cite relevant evidence (cases, legislation, literature and so on) in support of your argument. You will also need to deal with matters that appear, at least at first glance, to sit in opposition to your argument (on which see further below) or which, once properly considered, require your argument to be refined.  

A key point, however you proceed, is that the reader should also be clear about how each successive point relates not only to the previous point but also to the overarching argument. The reader should never be left wondering ‘Where does this fit in?’ or ‘Why am I being told this?’ A simple way of avoiding these problems is to signpost , by saying at the beginning of each section how it relates to the overall argument. The flipside of this coin is that you should avoid saying things like ‘Another point is that…’ since this gives the impression, rightly or wrongly, that the various points in your essay have been thrown together in a random order, with little thought as to how they fit together or relate to your overall argument. Even if that’s not the case, you don’t want to risk giving the reader that impression.

A one-sided approach?

The advice set about above might seem to imply that I’m suggesting you write one-sided essays — in which you set out points that support your argument while ignoring those that don’t. However, that’s not at all what I’m suggesting. In order to set out your argument in a persuasive manner, you need to deal both with relevant points that support your argument and with relevant points that appear to challenge your argument — and, in dealing with the latter points, you need to show why they do not in fact fatally undermine your argument. In other words, the approach I’m suggesting here doesn’t mean that you should adopt a blinkered approach, paying no attention to counterarguments: rather, you need to deal with them in a way that shows that, having thought about and weighed them in the balance, you are in a position to show why your argument stands in spite of them (or why your argument can be adapted in a way that accommodates such points).  

All of this points towards a further matter: namely, that advancing an argument in your essay does not mean that you need to (or should) be argumentative in the sense of adopting a strident tone that brooks no debate or compromise. Rather, advancing an argument in the way I’ve suggested here means being thoughtful and persuasive : taking the reader with you on a journey that demonstrates that you have looked at the relevant material, carefully thought through the issues raised by the question, and arrived at a view that you are able to justify and defend through well-reasoned and suitably evidenced argument.

So what about your conclusion? If you’ve followed my advice above, it should more or less write itself. People often agonise over conclusions, perhaps thinking that there has to be some ‘big reveal’ at the end of their essay. But there doesn’t need to be — and indeed there shouldn’t be — any big reveal. There should be no surprises at the end precisely because you’ve set out your argument at the beginning and spent the rest of the essay carefully constructing the different strands of your argument. The conclusion is an opportunity to draw those stands together, but no-one should have to wait with bated breath for the conclusion before finally realising: ‘Ah, so that’s what they think!’ If that’s the impact of the conclusion on your reader, it means there’s something wrong with the introduction!

This post was first published on The Law Prof blog . It is re-published here with permission and thanks.

' src=

  • Already have a WordPress.com account? Log in now.
  • Subscribe Subscribed
  • Copy shortlink
  • Report this content
  • View post in Reader
  • Manage subscriptions
  • Collapse this bar

How To Answer Law Questions (Essay & Problem Questions)

  • Post author: Edeh Samuel Chukwuemeka ACMC
  • Post published: July 4, 2020
  • Post category: Legal Articles / Scholarly Articles

How to answer law questions: Over the years, law schools and law universities have evolved two methods of asking test or exam questions. They are the problem and essay questions patterns. Basically, these two ways of asking questions require a totally different ways of answering them that are different from the traditional ways of answering questions in the primary, secondary, and tertiary institutions.

One major problem that new law university intakes encounter, is the fact that many of them usually have the mindset that where they are (university) will be the same with where they are coming from (secondary school); and because of this, they tend not to ask questions about how well to answer their law questions thus going on to use the traditional or general method and ending up not getting the expected result or even failing.

In this article, I am going to be expounding on the way of tackling law essay questions for the benefit of students. However, what I will be discussing is the general rule which is subject to what your specific tutor or lecturer may require from you.

So, it is advised that in as much as you learn how to generally answer law essay questions, you should pay attention and understand your lecturers so as to give him/her what their individual preferences are, like asking you to write a concised answer or be elaborate, or add this and remove that  etc.

Differences between cross-offers and counter-offers

Hierarchy of courts in Nigeria

Mojekwu v Mojekwu: Facts, Issues and Decision of the court

Table of Contents

Differences between Law problem and Essay Questions

As you may know, Law problem questions are not the same as essay questions. Problem questions are those law questions that tell live stories about the relationship between people and then require you to identify legal issues from those interactions, address the issues with relevant authorities and then to advise the parties on their different rights using the IRAC method.

It is important to note that a single problem question could have a series of different events which are based on different legal principles.

On the other hand, an essay question is a question which requires an individual to write on legal principles without using a particular format like the IRAC method.

Notminding the fact that you are not required to advise the parties or to use a particular method to answer your question, you have to have some mental steps at the back of your mind so as to make your work enticing and arranged before your tutor or Lecturer.

HOW TO ANSWER LAW PROBLEM QUESTIONS USING IRAC METHOD

Just like I explained at the beginning of this work, a problem question is a question that test candidates by giving them stories/cases to solve. Here, law students will be expected to draw out the issues of law in the story, as it relates to what they have been taught in the classroom.

Take for instance, a problem question on customary law might tell the story of a man who beats his wife because the custom provides for it. After explaining the story, students will be required to either provide the position of the law on the issues raised in the story or to advise the parties in the story.

The most acceptable way of answering law problem questions is through IRAC method. IRAC is actually the best because it makes it very easy for students to explain any legal principle with authorities in the simplest format. Below are the things you must know about IRAC method of answering law questions.

Meaning of IRAC

The word “IRAC” is an acronym which stands for:

  • Issues – I
  • Rule of law – R
  • Application – A
  • Conclusion – C

Note that if you must answer problem questions using this format, you must have this acronym at the back of your mind. If you miss any step, then you are getting the whole question wrong.

Now, to make sure that you understand the steps listed above, i will take my time to explain what you are supposed to do in every step. Remember, this is to teach you how to answer law problem question using IRAC method.

Your first concern is to determine the issue or issues implicated in the question. This determination involves asking yourself, what is the problem sought to be addressed in the case?

Example: What is the liability of a master regarding tort committed by his servant while engaged in a conduct expressly prohibited by the master?

The importance of accurate identification of the issue(s) is that it narrows your response to the gist of the question.

Once you accomplish this goal, you will know automatically that there is no need to state, for instance, that “the tort borders on vicarious liability” or for you to describe general elements of the tort in question.

Your task is to focus only on those elements or information that substantively (not tangentially) speak to the issue(s) you have successfully identified. Relevance is the key here. Recall my admonition, “ the more you write, the more you expose your ignorance .”

Note that you are not expected to call the names of parties in the story in the issues because they are issues for determination in law. You can only mention the names of parties in the story or case given to you when you reach third stage which is APPLICATION

Also read: Are lawyers liars? The truth as to whether lawyers are liars

RULE OF LAW

The “R” or Rule (rule of law) in IRAC is also called “reasoning.” This is because the applicable rule of law is reasoned from the facts of the case. As you think through the problems presented, aided by the issue you have successfully identified, the rule will emerge. What rule of law will guide the court in reaching a correct decision, assuming the same facts?

This is the question that you need to ask your self; it is also the question that you need to address. A rule of law in vicarious liability, for instance, is that “ a master is liable for the acts of his servant, even when expressly prohibited, so long as the servant acted within the scope of his employment. ”

Under the rule of law, students are expected to cite their authorities. Authorities here can be cases, statutes, dictum of judges, articles which are related to the issue in question. It is very important that you cite authorities because that is what will back up the rule of law and legal principles in the case.

APPLICATION:

This is where you apply the rule of law to the issue(s) you have raised. By doing this, you are applying the rule of law to the actual story in the problem question given to you. In the application, you are expected to pick those authorities and rules of law that concerns the issues raised and apply them to the matter effectively.

Always pay attention to exception(s) to the general rule, if any, and clearly outline/justify any distinctions that might be helpful to your argument. By constantly reminding yourself of the issue(s), you are bound to succeed in steering yourself away from irrelevance.

A great analysis is targeted to the issue(s) identified and is judged by the degree of focus/precision as well as the presentation (language/expressions used in articulating your argument).

Finally, the conclusion (“C”) . A few sentences would suffice to wrap up your discussion. Briefly state the outcome of your analysis. Where the question requires that you advise the parties, the conclusion is the best place to do that.

Simply tell each of the parties their rights and persuade them to sue the when the need be. Here, you can also rebuke the party in default in the case and tell him why he/she is at fault.

How to answer law problem question

Okay! Now that you know what all the letters in the acronym IRAC entails, I will give you an example of how a perfect law problem question is answered. The example below is a problem question that borders on the law of defamation.

If you are a student and you don’t know anything about the law of defamation, don’t worry. Just keep reading. You will still be able to understand the steps taken in every section.

Also read: List of less competitive universities in Nigeria (2020)

Sample of a law problem question answered using IRAC method

The Daily Trumpeter, a popular Newspaper in Enugu recently published a report of the proceedings of the Enugu State High Court in a land case between Chief Okoto and Barrister Akuepue under the caption ‘Judge calls a popular Enugu Lawyer: ‘A Crook and a Land Speculator”.

In the article, the newspaper reporter, Ade also stated thet Nigerian lawyers are in the habit of using their knowledge of the law to deprive innocent ‘laymen’ of their land. Barrister Akuepue and Barrister Ikpeama, another popular lawyer based in Enugu have sepaprately sued Joe, the edito of The Daily Trumpeter and Ade for publication.

Issue 1: Whether Newspaper Publishers can be liable for publishing court/tribunal proceedings

Issue 2: What must one prove in order for his defense of ‘fair comment’ to be successful

Issue 3: What is the legal position on defamation of a class or a group of person

RULE OF LAW:

Defamation refers to the publication of a statement which is calculated to injure a person and cause right-thinking members of the society to shun or avoid him, or even cause them to hate him and also convey an imputation on him which is injurious to his office, trade or profession – s.137 Enugu State Torts law, cap 150 2004(which shall hereinafter be called ESTL), Sketch v. Ajagbemokeferi.

People are thus warned against idle gossip which may likely impugn another person when communicated to a third party. Defamation may either be in the form of libel which is in a permanent form such as newspaper publication, television or radio broadcasts; or otherwise in slander which has a transient nature usually verbalized or through gesticulations conveying a defamatory connotation.

For an action in defamation to succeed, the following essential elements must be proved, as a thing of necessity:

The words complained of must be defamatory:

If right-thinking people of sane minds would think less of an individual or shun and avoid him due to a statement, then this element may be said to have been successfully proved.

Defamatory words must refer to the plaintiff:

It is not sufficient that the defamatory statement described a person merely by his name Akintla v. Anyiam . It is enough where he is identified by his initials, post, Photograph, or even his office – Dafe v. Teswinor.

The words complained of must be published:

It has been held in a vast litany of cases that it is not the publication of defamatory statement but the publication that grounds a cause of action. In fact, in Pullman v. Hill , Lord Esher, Master of the Rolls said thus ‘‘ Publication is the making known of the defamatory matter after it has been written to such person other than the person to whom it is written’’ It therefore follows that publication in itself is what grounds a cause of action- s. 141 ESTL .

Also, communication to the plaintiff himself cannot ground a cause of action for the purposes of determining liability in defamation because defamation is injury to ones reputation and protects not an individual’s opinion of himself but the estimation in which others hold him – Okotcha v. Olumese.

However, even if a person has been alleged to have defamed another, there is an array of defenses open to him. Such defenses are:

  • Unintentional Defamation
  • Innocent dissemination
  • Justification( or truth) – s. 163 ESTL
  • Volenti Non fit injuria – Chapman v.
  • Fair comment- s. 194(1) ESTL

Under the defenses of privilege , we have what is known as absolute privileges and also qualified privileges. Circumstances under which the defense of qualified privilege can arise are varied but for the purposes of our case, we have an occasion known as Statements made in performance of a legal, moral or social duty – s.178 ESTL .

In the connection above, for such an occasion to arise, the person giving out information which is alleged to contain defamatory statements must have a duty to give such information on grounds of public policy and also the party receiving such information, that is, the person to whom the matter is published to must have a corresponding duty of receiving such information.

Perhaps, this is the reason the law admits that radio and television broadcasters as well as Newspaper Publishers and Proprietors are covered by the said defense – NTA v. Babatope . It is also the legal position, pursuant to s.185 (1) that qualified privilege very much applies to those who publish reports of judicial proceedings. However, such broadcasters or publishers must be very careful enough to give reports of what actually took place in court, not necessarily a verbatim report of the proceedings but at least an abridged or condensed report will be privileged, provided that it gives a fair, accurate and correct impression of what transpired.

Also read: Names of Nigerian Presidents From 1960 till Date (Full list) 

It clearly and necessarily follows that when such reports are substantially inaccurate, such a report will lose the protective cloak of this defense. Thus, in Omo-Osagie v, Okutobo, a report of a newspaper of certain court proceedings bore the caption, ‘‘Chief Justice Tells a Teacher: ‘ You are a Bad Woman’.

However, those words were never used by the Judge, and the courts held that such a newspaper report had lost the defense of qualified privilege. The defense of fair comment stated above, consists of criticisms of matters of public interest in the form of comments, by citizens upon true facts, such comments being honestly made without malice – s.194(1) Enugu State Torts law, cap 150 2004.

In order for this defense to avail a person, the following requirements must be proved to the satisfaction of the courts trying the issue:

The Matter commented on must be of Public Interest:

The matter must be one of general concern as to affect the generality of the population at large- London Artists Ltd. v. Littler. Thus issues of land fraud by legal practitioners may fall for issues of public interest.

The Comment must be an expression of Opinion not an assertion of fact:

It is noteworthy that the defense of fair comment consists of two things: a set of facts which must be true and the commentator’s opinion on those facts- s. 194(1) (a). The distinction between a comment and a fact, however, depends on the merits of each case.

The Comment must be Honest- s. 194(1)(c) So long as a commentator honestly expressed his view, it is immaterial that he used excessively strong language o that people read all sorts of innuendoes into it, if he made the comment honestly, he has nothing at all to fear. However, criticisms cannot be used as a cloak for an attack, or for personal imputations on the plaintiff not arising out of the subject matter not based on the facts.

The Comment must be devoid of Malice- s.195 ESTL Malice is a complete bar to a defense of fair comment. Malice means making abuse of the occasion for some indirect purpose – Bakare v. Ibrahim.

When a class or group of persons i.e. lawyers, teachers, doctors etc, are defamed, no individual member of the class is entitled to bring action upon grounds that he has been defamed save only when the class is so small or so ascertainable that what is said of the class is necessarily said of each and every member of it, or if the circumstances of the case show that the plaintiff was singled out and defamed – Knuppfer v. London Express Newspaper Ltd.

Thus, in Zik Enterprises ltd. V. Awolowo , an article which contained defamatory statements against the Action Group was regarded by the court not to refer to the plaintiff but to the Action Group as a political party since it was a large group and the plaintiff could not show circumstances which proved that he was singled out.

Also read: How to answer Law problem questions effectively

APPLICATION

It seems that Daily Trumpeter will not escape liability if the report they gave of the court proceedings was grossly inaccurate or did not give a correct impression of what actually transpired therein.

Also, Ade seems to have overstepped the boundaries of fair comment by making such a comment, he seems to have made a statement of fact because saying that someone is in the habit of doing something is as much as imputing a disreputable motive upon him which is very much actionable in law. Hence the cloak of fair comment may no longer protect.

Barrister Akuepue and Barrister Ikpeama by bringing action against Ade seem to be oblivious of the applicable guiding principles in the circumstance. In this connection, a defamatory statement against Nigerian lawyers as a whole is too large a group for the purposes of an action in defamation, unless, if they can show that they were singled out and defamed. They may be going on a wild goose chase.

Joe : You will only escape liability if what you said happened in court was accurate, otherwise, you will fall for liability. Ade: It is true you have overstepped the bounds of fair comment; however you may still escape liability under the principles of defamation of a class or group.

Barrister Akuepue and Barrister Ikpeama : You both can only bring action against Ade and Joe should what Joe reported about the proceedings be inaccurate. With respect to Ade although he has lost the defense of fair comment, he can still be liable for publishing what Joe reported, only the will you have a cause of action against him.

Must read: Most populated universities in Nigeria (2020)

Four things to note when answering law questions

How to answer law problem question

Note the little differences in the way every lecturer want students to answer their questions:

Even though IRAC method of answer law problem questions is generally acceptable, some lecturers make little changes to the way they want their questions to be answered.

For instance, some lecturers posit that after outlining your issues, you must tackle them one after the other. What that means is that, you are expected to apply IRAC in the different issues ( One after the other ).

Well, it is impossible for me to cover all the different styles lecturers like. So, I enjoin you to always attend classes. By attending classes, you will know the best method to answer your law problem questions.

It will surprise you to know that some lecturers don’t even like their questions to be answered using IRAC. So you see, you must know what every lecturer wants.

Time is a very important factor:

To perform very well as a law student, you must have it in mind that time is very important. You have to be time conscious because you have only but 2:30 minutes to answer all the questions you were given. Most times, students are expected to answer at least 3 questions in law examinations.

So you must learn to manage your time. If you do not manage your time properly, you will definitely not finish answering your questions.

How to answer law problem question

I personally recommend that you get a wristwatch for your exams. It will go a long way to help you plan how to use your time adequately.

Always cite your authorities with red pen:

One of the pivotal information that has never been skipped in all the tutorials I have read on how to answer law problem question is the citing of authorities.

The importance of using a red pen when citing authorities in law examinations cannot be over emphasized. The reason is because, it makes it easy for anyone who is assigned to mark your examination script.

So, in other not to jeopardize your chance of success in any law exam, you should use red pen to cite your authorities. Your authorities here may include: cases, names of judges, articles, statutes, dictums etc.

Dive detailed information about the issues raised in every case:

Most times, lecturers prefer giving more marks to students who were able to give them detailed information about the question than students who just answered the question.

For sure, if you answer the question correctly you will definitely be given the mark you deserve. But if you give more details, you will likely earn more marks.

Take for instance, you might be asked to explain the term “ Nigeria legal system “. If you go ahead to just define it and move to the next question, you won’t even get your full marks there. To get your full marks, you must also highlight the features of Nigerian legal systems and any other subtopic in the topic. Though, you are not expected to go too deep. Just make sure you give detailed information. Some lecturers love it.

Okay! for now, this is all i can disclose on this topic (How to answer law problem question). Like i have rightly mentioned, it is important to always attend classes because the way a particular lecturer will want his/her problem question to be answered might be different from the way another lecturer wants it.

That notwithstanding, i have explained the most generally accepted way of answering law question using IRAC method above. So, if you have not written law examination before, you can safely understand the IRAC method as i explained it. Hope this article was helpful? Do let me know if you have any question or confusion as to how to answer law problem question using IRAC method. I will be glad to help.

how to answer a law essay question

Edeh Samuel Chukwuemeka, ACMC, is a lawyer and a certified mediator/conciliator in Nigeria. He is also a developer with knowledge in various programming languages. Samuel is determined to leverage his skills in technology, SEO, and legal practice to revolutionize the legal profession worldwide by creating web and mobile applications that simplify legal research. Sam is also passionate about educating and providing valuable information to people.

A-Level Law

The right page you need to master a-level law., created by a student for the use of other students 😉.

Improve your knowledge with out materials, practice exercises, games… and more!

Welcome to the the A-Level Law website!

Simple and easy to use. Oh, and colourful!

Struggle to answer A-Level Law essay questions? Check the I.D.E.A.L. method of tackling them.

Practice makes perfect!

Law is impossible without knowing your terms.

Learn and apply cases in support of your arguments.

University or apprenticeship? Find out some advice.

Find out who stands behind the website.

Edmund Burke

‘Laws, like houses, lean on one another.’

William E. Gladstone

‘Justice delayed is justice denied.’

William Godwin

‘Justice is the sum of all moral duty.’

Thomas Jefferson

‘A lawyer without books would be like a workman without tools.’

Share this website on social media. Help grow our international community of users! 🙂

Our website has visitors from 80 locations around the globe, including:

how to answer a law essay question

What do you think of the website? Tell us below by either writing a comment, or filling in our form…

https://forms.microsoft.com/Pages/ResponsePage.aspx?id=HhF8To_Dg0GAnm-GuB7UZA4Hq6JiHjVHiEwAhW4qUWpUMlRWUFdXMEozMVIwNjgxOUc1NFpOUVNGWC4u

Our rating (out of 5.0):

how to answer a law essay question

Brief customer review:

how to answer a law essay question

Share this:

10 thoughts on “ a-level law ”.

I love you my man. LEGEND

Like Liked by 1 person

Thank you very much, Jake! Our users’ support is really appreciated!

Very useful website, made revision a lot easier!

Thank you, Alannah! It is much appreciated!

Very practical and useful website. No negative aspects at least for me!

Thank you, Grisha! It is much appreciated!

Well, it is made by this guy… So you know it is 10/10. If there is someone you can trust about law, it is literally this guy. Great website, great content, easy and simple to understand.

Your review is extremely appreciated! Thank you very much for the support! 🙂

Congratulations from Brussels, Georgi!!! I’m proud of you!!! You are the man !!!

Thank you so much, Kalin! Really kind and inspiring words! 😊

Leave a comment Cancel reply

' src=

  • Already have a WordPress.com account? Log in now.
  • Subscribe Subscribed
  • Copy shortlink
  • Report this content
  • View post in Reader
  • Manage subscriptions
  • Collapse this bar

Putting the 'memorial' in Memorial Day: How the holiday evolved over time

how to answer a law essay question

Though it falls nearly a month before summer actually begins, many consider Memorial Day as the unofficial kickoff to summer. On May 27, folks will be firing up the grill and enjoying a day off work, but the holiday has “memorial” in its name for a reason.

The first national observance of Memorial Day came courtesy of Gen. John A. Logan , commander in chief of the Grand Army of the Republic. On May 5, 1868, Logan issued General Order No. 11 calling for a national day of remembrance for Union soldiers killed in action:

"The 30th day of May, 1868 is designated for the purpose of strewing with flowers or otherwise decorating the graves of comrades who died in defense of their country during the late rebellion... (II) It is the purpose of the commander in chief to inaugurate this observance with the hope that it will be kept up from year to year..."

The new holiday was called Decoration Day and it is believed that May 30 was chosen because flowers would be in bloom across the nation.

More: How many federal holidays are there? Make sure you don't miss your days off in 2024-25

Need a break? Play the USA TODAY Daily Crossword Puzzle.

Honoring soldiers killed in other American wars wasn’t commonplace until after World War I. Also, the name Memorial Day became more widely used than Decoration Day after World War II, but wasn’t declared the official name by federal law until 1967.

On June 28, 1968, Congress passed the Uniform Monday Holiday Act, moving Memorial Day from its traditional date to the last Monday in May and creating a three-day weekend for federal employees. The law took effect at the federal level in 1971.

To guarantee that the sacrifices of America’s fallen heroes are never forgotten, Congress passed and President Bill Clinton signed into law the National Moment of Remembrance Act in 2000. On Memorial Day, Americans are urged to pause at 3 p.m. for a minute of silence to remember those who died serving their country.

[email protected]

  • Find a Lawyer
  • Ask a Lawyer
  • Research the Law
  • Law Schools
  • Laws & Regs
  • Newsletters
  • Justia Connect
  • Pro Membership
  • Basic Membership
  • Justia Lawyer Directory
  • Platinum Placements
  • Gold Placements
  • Justia Elevate
  • Justia Amplify
  • PPC Management
  • Google Business Profile
  • Social Media
  • Justia Onward Blog

Q: How should I file tax if my employer don’t issue me a W-2 since I’m a part time intern? I make hourly.

I got approved CPT by the school before started working. So everything is legal but I’m just worried about how should I file tax to avoid futures consequences.

James L. Arrasmith

  • Immigration Law Lawyer
  • Sacramento, CA
  • (916) 704-3009
  • Email Lawyer
  • View Website

A: If your employer doesn't issue a W-2 because you're a part-time intern, you might be classified as an independent contractor. In this case, you should receive a Form 1099-NEC if your earnings exceed $600 in a year. Even if you don’t receive this form, you’re still responsible for reporting your income to the IRS. To file your taxes, gather all your pay stubs or any records of the income you've earned. You'll need to report this income on your tax return using Schedule C (Profit or Loss from Business) along with Form 1040. Remember to also account for any expenses related to your work, as these can be deducted to reduce your taxable income. If you have any questions or feel uncertain about the process, consider consulting a tax professional who can provide guidance tailored to your situation. This will help ensure you comply with tax laws and avoid any potential issues in the future.

Justia Ask a Lawyer is a forum for consumers to get answers to basic legal questions. Any information sent through Justia Ask a Lawyer is not secure and is done so on a non-confidential basis only.

The use of this website to ask questions or receive answers does not create an attorney–client relationship between you and Justia, or between you and any attorney who receives your information or responds to your questions, nor is it intended to create such a relationship. Additionally, no responses on this forum constitute legal advice, which must be tailored to the specific circumstances of each case. You should not act upon information provided in Justia Ask a Lawyer without seeking professional counsel from an attorney admitted or authorized to practice in your jurisdiction. Justia assumes no responsibility to any person who relies on information contained on or received through this site and disclaims all liability in respect to such information.

Justia cannot guarantee that the information on this website (including any legal information provided by an attorney through this service) is accurate, complete, or up-to-date. While we intend to make every attempt to keep the information on this site current, the owners of and contributors to this site make no claims, promises or guarantees about the accuracy, completeness or adequacy of the information contained in or linked to from this site.

  • Bankruptcy Lawyers
  • Business Lawyers
  • Criminal Lawyers
  • Employment Lawyers
  • Estate Planning Lawyers
  • Family Lawyers
  • Personal Injury Lawyers
  • Estate Planning
  • Personal Injury
  • Business Formation
  • Business Operations
  • Intellectual Property
  • International Trade
  • Real Estate
  • Financial Aid
  • Course Outlines
  • Law Journals
  • US Constitution
  • Regulations
  • Supreme Court
  • Circuit Courts
  • District Courts
  • Dockets & Filings
  • State Constitutions
  • State Codes
  • State Case Law
  • Legal Blogs
  • Business Forms
  • Product Recalls
  • Justia Connect Membership
  • Justia Premium Placements
  • Justia Elevate (SEO, Websites)
  • Justia Amplify (PPC, GBP)
  • Testimonials

IMAGES

  1. 50 Important Law Question and Answers

    how to answer a law essay question

  2. Introduction to Law Essay

    how to answer a law essay question

  3. How to Write a Law Essay

    how to answer a law essay question

  4. 😍 How to write an essay question. How to Effectively Answer Law Essay

    how to answer a law essay question

  5. How Do You Answer An Essay Question

    how to answer a law essay question

  6. How To Write a Good Law Essay?

    how to answer a law essay question

VIDEO

  1. Charles's law, essay questions

  2. UCC CONTRACT LAW ESSAY QUESTION

  3. LAT (Law Admission Test Preparation)

  4. Important problems in Current Electricity Lesson for Intermediate II Year Linked problems to essay

  5. Law Writing Practice

  6. Employment Law in Detail: An Overview

COMMENTS

  1. How to Effectively Answer Law Essay Questions

    Answering Law Essay Questions Rule 1: The Introduction. The introduction to your law essay question is the part where you let the lecturer know what the answer is all about. In this part of the question, you shouldn't directly go into answering the question. Instead, you are allowed to beat about the bush a little bit.

  2. How to Write Law Essay Questions

    The number one rule of answering any law essay question is RTFQ - ATFQ. Read the flaming question, answer the flaming question. Here is a common complaint by law examiners every year. Many students read the question once, see its on a particular topic, and regurgitate everything they know about that topic.

  3. Interpreting Law Essay Questions

    Share this: Interpreting Law Essay Questions. The first step with a law essay question is to identify what exactly you are being asked to do. Most law essay questions contain directives as to what is required, and the most common ones are defined below. For those we have missed, try looking up the actual word in the dictionary (www.m-w.com).

  4. PDF Law School Guide to Answering Essay Questions

    The purpose of this guidance. This guide is intended to serve a baseline level of guidance for approaching essay questions in any subject offered by the Law School as part of your undergraduate law degree. It cannot, and does not purport to be, exhaustive. It is structured in a way which reflects the essay writing process and the main parts of ...

  5. How to Structure a Law Essay (Tips from a Former LLB Lecturer)

    Restate key supporting arguments. The final stage of creating the plan of your law essay is to pick 2 to 3 key supporting arguments which you discussed in the main body of your paper and outline them again. This time, however, you will not be getting into a detailed discussion of how case law or statute sections justify your supporting arguments.

  6. How to Answer Law Essay Questions (Law Lecturer's Guide)

    Unfortunately, law lecturers are quite busy people and they don't really have time to spend on training you how to answer law essay questions. The 1stClassLLB Templates that I have created, basically contain a formula that applies to all law essays, no matter the module, for both essay-style and problem-style questions.

  7. Answering Essay and Problem Questions

    The best approach to answering problem questions is the IRAC method, which stands for issue, rule, application, and conclusion. This is a four-step process that requires you to identify the legal issue; identify the relevant legal rule; apply that rule to the issue at hand; and reach a conclusion to answer the question.

  8. Writing Law Problem Questions

    This guide will explain how to answer a problem essay with eight handy tips. 1. Read the Facts. The first step to answering any law problem question is to read the entirety of the facts you are given. Do not just jump into answering the question. Take your time and ensure that you fully understand all the issue involved in the case.

  9. Law: Legal essay

    There are a number of strategies that may help you in starting, structuring and presenting a law essay. 1. Starting your answer. The first step to a successful law essay is understanding the question. One of the most effective ways of breaking down the question is to identify the direction, content, and scope or limiting words.

  10. How to "Critically Evaluate" and "Analyse" in Law Essays

    In order to critically evaluate a law essay, you need to go beyond asking the basic questions. Just answering with the specific law on a certain issue isn't enough to make you a good lawyer. You need to build the ability to think for yourself and have an opinion on every case and statute, which you can defend with solid arguments.

  11. Structure Of Law Essays and Reports

    Body:The body of your answer, accounting for the majority of the word count, should demonstrate your understanding of the area and develop your argument. It is a good idea here to keep referring explicitly to the question asked. Conclusion: The conclusion for essay style questions will represent about 10 - 15% of your word count. This must ...

  12. How to Answer Law Essay Questions

    If you're struggling to answer questions in your law essays, the advice in this video will point you in the right direction.-----...

  13. How do I answer law school exam questions? (an in-depth guide)

    Here, we give you an in-depth guide on how to answer law school exam questions. This is a step-by-step process for approaching law school exams. Law school exams are very different than exams you may have taken in undergrad. Because they are different from the exams you are used to, you need a novel strategy if you want to answer the law school ...

  14. Problem Question Help Guide

    Free Law Problem Question Answer Guide. Example of how to answer problem questions from the experts at Law Teacher. ... Essays, case summaries, problem questions and dissertations here are relevant to law students from the United Kingdom and Great Britain, as well as students wishing to learn more about the UK legal system from overseas.

  15. Writing a Law essay? Remember to argue!

    Nevertheless, a useful rule of thumb, I think, is that a good Law essay will normally set out and advance a clear thesis or argument. (Note that I'm referring here to essays as distinct from problem questions: the latter call for a different approach.) The need for an argument. Some answers explicitly call for this.

  16. PDF HINTS AND TIPS ON ANSWERING EXAM QUESTIONS

    Essay questions typically give you an opportunity to engage critically with contemporary issues in criminal law. Essay questions expect you to go beyond mere description of the law and demonstrate an awareness of: - why the law is as it is; - why the law may be controversial; - how the controversies relate to overarching and pervasive principles of

  17. PDF How to Answer Fact Based Law Exam Questions

    T ypes of Examination Questions There are several types of law exam questions: short answer , essay questions, fact based questions, etc. Short answer questions typically require a few sentences by way of responses and assess recall of information. Essay questions test a candidateÕ s familiarity with and an ability to

  18. How To Answer Law Questions (Essay & Problem Questions)

    How to answer law questions: Over the years, law schools and law universities have evolved two methods of asking test or exam questions. They are the problem and essay questions patterns. Basically, these two ways of asking questions require a totally different ways of answering them that are different from the traditional ways of answering questions in the primary, secondary, and tertiary ...

  19. PDF How to Answer Problem Questions

    Heres the truth: problem questions are long. An essay question might only be a couple of lines but a problem question can often run for an entire page of A4. In an exam when it is you vs. the clock this can be an intimidating prospect and, to save on time, a lot of students will just dive straight in and start writing. That is a false economy.

  20. A-Level Law

    Your future with Law. About the creator. Welcome to the the A-Level Law website! Simple and easy to use. Oh, and colourful! Struggle to answer A-Level Law essay questions? Check the I.D.E.A.L. method of tackling them. Practice makes perfect! Law is impossible without knowing your terms.

  21. How can I have a dismissed civil record removed?

    The use of this website to ask questions or receive answers does not create an attorney-client relationship between you and Justia, or between you and any attorney who receives your information or responds to your questions, nor is it intended to create such a relationship.

  22. Questions and Answers: New EU Law on Corporate Value Chains

    This question-and-answer document reviews the provisions, strengths, and weaknesses of the new EU Corporate Sustainability Due Diligence Directive, adopted on May 24, 2024.

  23. Not working on probation

    James L. Arrasmith. Answered 2 hours ago. Criminal Law Lawyer. Sacramento, CA. Licensed in California. (916) 704-3009. Email Lawyer. View Website. A: Under California law, the requirement to find gainful employment as a condition of your probation can be flexible, especially if your probation officer is aware of and has not objected to your ...

  24. I have received a document in the mail "Notice of decision to award

    Read 1 Answer from lawyers to I have received a document in the mail "Notice of decision to award $1,000,000.00" and "Prize payout decision Request" - South Carolina Consumer Law Questions & Answers - Justia Ask a Lawyer

  25. Law Essay Help Guides

    The law essay help guides below were written by our expert writers, as a learning aid to help you with your studies. If you are looking for help with your law essay help guide then we offer a comprehensive writing service provided by fully qualified academics in your field of study. Law Essay Writing Service.

  26. My father signed a new will several times. We were unable to have it

    A: I am very sorry for father's illness and his health struggles ongoing. There will be a serious issue with such a Will in Florida, it will not be a valid Will as it does not meet the Florida Statutory requirements of F.S. 732.502 (you can look it up online), what you have described without any witnesses is a Holographic Will, Florida will not accept them or recognize them, the original Will ...

  27. How, why and when Memorial Day began as a national holiday in 1868

    The first national observance of Memorial Day came courtesy of Gen. John A. Logan, commander in chief of the Grand Army of the Republic. On May 5, 1868, Logan issued General Order No. 11 calling ...

  28. Do I have the right to remove my child from her mother's cate if her

    A: Under California law, the primary concern in child custody cases is the best interests of the child. While your ex's association with a gang member who is facing criminal charges is concerning, you generally cannot unilaterally remove your child from the mother's care without a court order.

  29. EU Adopts Groundbreaking Business Value Chain Law

    Human Rights Watch issued a question-and-answer document about the provisions, strengths, and weaknesses of the new law. The document describes how the EU Corporate Sustainability Due Diligence ...

  30. How should I file tax if my employer don't issue me a W-2 since I'm a

    A: If your employer doesn't issue a W-2 because you're a part-time intern, you might be classified as an independent contractor. In this case, you should receive a Form 1099-NEC if your earnings exceed $600 in a year.